Argument Structure Questions - - Question 14

The government has spent heavily to clean groundwater contaminated by toxic chemical spills. Yet not even one spill ...

yababio May 21, 2015

help

can someone break this down for me in detail i mean break down the paragraph the question all the answer choices everything step by step because im tired of reading a explanation and i still dont get it im really this close for asking for a refund because you guys claim your better than kaplan and powerscore yall are just like them its annoying im a visual learner i need visual stuff to learn.

Replies
Create a free account to read and take part in forum discussions.

Already have an account? log in

Naz May 21, 2015

I'm sorry about your frustration, but this is an Argument Structure question. There is nothing visual about this question. It would literally just be you listening to me read the question and identify the conclusion and premise. If you have any issues on questions that actually have a majority of visual components, then we would be happy to make a video for you.

This is much better just written down, i.e. Conclusion: More of the government's budget should be redirected to preventing spills; Premise: Despite money being put in to clean up spills, none of the spills have been completely cleaned and more toxic chemicals are spilled annually than are being cleaned up and we are told that prevention is far more effective than cleanup.We also know that the entire annual budget for prevention is less than the amount spent annually on one typical cleanup site.

The question stem asks us to identify the role played by the statement on how the government's budget should be redirected. Well, as we identified above, that is our conclusion. So, clearly the correct answer choice is (C) since it describes the role of a conclusion in an argument: "It is the claim that the argument as a whole is structured to support."

We know from the video lesson that a conclusion is what the rest of the argument supports. Thus, answer choice (C) is the correct answer.

Hope that clears things up! Please let us know if you have any other questions.

ddennehy7 March 14, 2018

My only confusion with this specific question is the issue I have with the word "support" in the answer choice C. You guys have reiterated that a conclusion cannot support anything else in the argument. I originally wanted to choose C as my answer but I changed it for that very reason. Maybe I am simply overthinking it.....?

Mehran March 14, 2018

A conclusion does not support other parts of the argument, but the other parts of the argument will support the conclusion so you should absolutely not eliminate an answer choice on an Argument Structure question solely because the word "support" appears in it.

Look at the language of (C), "It is the claim that the argument as a whole is structured to support."

The conclusion is the claim that an argument as a whole is structured to support.

So (C) would be the correct answer here.

Hope that helps! Please let us know if you have any other questions.

edward101099 June 2, 2019

Thank you for your clear and concise explanation! The lesson plan guided me to the right answer but I read the explanation just in case. LSAT MAX has the BEST instructors ever.

Ravi June 2, 2019

@edward101099, happy to hear that! Let us know if you have any other questions—we're here to help!

Justitia2020 February 13, 2020

Thanks a million for breaking these questions down in presenting these explanations!!! I'm starting to get it!!!!